Question

In: Advanced Math

Prove that the rational numbers do not satisfy the least upper bound axiom. In particular, if...

Prove that the rational numbers do not satisfy the least upper bound axiom. In particular, if a subset (S) of the rational numbers is bounded above and M is the set of all rational upper bounds of S, then M may not have a least element.

Solutions

Expert Solution


Related Solutions

Recall the following theorem, phrased in terms of least upper bounds. Theorem (The Least Upper Bound...
Recall the following theorem, phrased in terms of least upper bounds. Theorem (The Least Upper Bound Property of R). Every nonempty subset of R that has an upper bound has a least upper bound. A consequence of the Least Upper Bound Property of R is the Archimedean Property. Theorem (Archimedean Property of R). For any x; y 2 R, if x > 0, then there exists n 2 N so that nx > y. Prove the following statements by using...
1. State the prove The Density Theorem for Rational Numbers.
  Question 1. State the prove The Density Theorem for Rational Numbers. Question 2. Prove that irrational numbers are dense in the set of real numbers. Question 3. Prove that rational numbers are countable Question 4. Prove that real numbers are uncountable Question 5. Prove that square root of 2 is irrational
1. State the prove The Density Theorem for Rational Numbers.
  Question 1. State the prove The Density Theorem for Rational Numbers. Question 2. Prove that irrational numbers are dense in the set of real numbers. Question 3. Prove that rational numbers are countable Question 4. Prove that real numbers are uncountable Question 5. Prove that square root of 2 is irrational
PROOFS: 1. State the prove The Density Theorem for Rational Numbers 2. Prove that irrational numbers are dense in the set of real numbers
  PROOFS: 1. State the prove The Density Theorem for Rational Numbers 2. Prove that irrational numbers are dense in the set of real numbers 3. Prove that rational numbers are countable 4. Prove that real numbers are uncountable 5. Prove that square root of 2 is irrational
let x:=7 show that x is the least upper bound of [3,7] show that x is...
let x:=7 show that x is the least upper bound of [3,7] show that x is the least upper bound of (3,7)
Let a and b be rational numbers. As always, prove your answers. (a) For which choices...
Let a and b be rational numbers. As always, prove your answers. (a) For which choices of a, b is there a rational number x such that ax = b? (b) For which choices of a, b is there exactly one rational number x such that ax = b?
Use the Well-Ordering Principle of the natural numbers to prove that every positive rational number x...
Use the Well-Ordering Principle of the natural numbers to prove that every positive rational number x can be expressed as a fraction x = a/b where a and b are postive integers with no common factor.
Prove that Poisson's weak law is a particular case of Chebyshev's weak law of large numbers.
Prove that Poisson's weak law is a particular case of Chebyshev's weak law of large numbers.
Prove that the real numbers do not have cardinality using Cantor’s diagonalization argument.
Prove that the real numbers do not have cardinality using Cantor’s diagonalization argument.
Prove that the real numbers do not have cardinality N0 using Cantor’s diagonalization argument.
Prove that the real numbers do not have cardinality N0 using Cantor’s diagonalization argument.
ADVERTISEMENT
ADVERTISEMENT
ADVERTISEMENT